Prepare for the LSAT with our comprehensive quiz featuring multiple choice questions, hints, and detailed explanations. This is your ultimate guide to mastering the LSAT and boosting your confidence on test day!

Each practice test/flash card set has 50 randomly selected questions from a bank of over 500. You'll get a new set of questions each time!

Practice this question and more.


Each of the following, if true, weakens the argument EXCEPT:

  1. Weakening X

  2. Strengthening X

  3. Weaken

  4. Strengthen Y

The correct answer is: Weaken

In this question, the task is to find which statement does not weaken the argument. Looking at the choices, options A, B, and D involve weakening or strengthening an argument. On the other hand, option C simply states "Weaken," which does not specify whether it weakens or strengthens the argument. Therefore, option C does not clearly weaken the argument like the other choices do, making it the correct answer as it does not align with the task of weakening the argument.